1answer.
Ask question
Login Signup
Ask question
All categories
  • English
  • Mathematics
  • Social Studies
  • Business
  • History
  • Health
  • Geography
  • Biology
  • Physics
  • Chemistry
  • Computers and Technology
  • Arts
  • World Languages
  • Spanish
  • French
  • German
  • Advanced Placement (AP)
  • SAT
  • Medicine
  • Law
  • Engineering
mr_godi [17]
3 years ago
14

Select the property of equality used to arrive at the conclusion

Mathematics
2 answers:
andre [41]3 years ago
8 0

Answer:

the answer is addition property of eqaulity

Step-by-step explanation:

u wtach james charles

aksik [14]3 years ago
6 0

Answer:

The addition property of equality

Step-by-step explanation:

2 + 8 = 10

10 = 10

You might be interested in
I need the answer.Help me out
rosijanka [135]

The answer is \boxed{7}, where a=5 and b=2.

<em>Solution sketch:</em>

Apply the Pythagorean theorem to find EC and AE, which you then use to find AD and BC. Apply Heron's formula to \triangle ABC to find that its area is 10, and then use [ABC]=\frac{(AB)(CF)}{2} (base times height over 2) to finish by solving for CF..

6 0
4 years ago
HELP<br><br><br> PLSS <br><br><br> DUE RNN<br><br><br> WILL GIVE BRAINLIST
Ne4ueva [31]

Answer:

Hello! answer 30

Step-by-step explanation:

It shows you right there when you connect x z and y you get 30 degree measure

4 0
3 years ago
Write True or False for the statement below
dusya [7]

Answer:

true

since: a = 1, or >1 and <10

7 0
3 years ago
What is the total investment of $2150 invested at 5.3% for 16 years and compounded continuously?
sladkih [1.3K]

Answer:

The total investment is $5020.19

Step-by-step explanation:

Given : $2150 invested at 5.3% for 16 years and compounded continuously

To find : What is the total investment?

Solution :

The formula of continuously compounded interest is

A = Pe^{rt}

Where A = amount invested

P = principal = $2150

e = Euler's number (on calculator)

r = interest rate as decimal = 5.3%=0.053

t = time in years = 16 years

Substitute the value in the formula,

A = Pe^{rt}

A = 2150\times e^{0.053\times 16}

A = 2150\times e^{0.848}

A = 2150\times 2.3349

A = 5020.19

Therefore, Option 4 is correct.

The total investment is $5020.19.

6 0
3 years ago
PLEASE HELP FAST!! <br>1. Find x. Show your work.​
Taya2010 [7]

Answer:

X is right there

Step-by-step explanation:

See?

7 0
3 years ago
Other questions:
  • The size of a television is the length of the diagonal of its screen in inches. The aspect ratio of the screens of older televis
    12·1 answer
  • HELP ASAP!!find the midpoint of the line segment MN, where M is (16,-4) and N is (2,10).
    7·1 answer
  • Find the quadratic equation for y=x^2+6x-7
    11·1 answer
  • Which of the following regression models is used to model a nonlinear relationship between the independent and dependent variabl
    11·2 answers
  • Find the x-intercept of the graph of each equation.<br><br> a. 3x+2y=7<br> b. 2x+3y=7
    6·2 answers
  • Solve for x. Your answer must be simplified.<br> -3
    12·1 answer
  • Which angles of the triangle measure 90 degrees?<br><br> I just need clarification
    15·1 answer
  • 6.7.37
    15·1 answer
  • Write 16+32 as a product of two factors using the GCT and 5e distributive property
    14·1 answer
  • Which of the following functions is graphed below?
    6·1 answer
Add answer
Login
Not registered? Fast signup
Signup
Login Signup
Ask question!